Re: [obm-l] OBMEP 2021 - Fase 2 - N3

2021-11-09 Por tôpico Matheus Bezerra Luna
Não é completamente reversível não, vai ter que usar o item C para concluir o D. Se num tempo T o ponteiro está em uma cara, no tempo T-1 ele poderia estar tanto numa cara (pois então nesse tempo não aconteceu nada e a moeda seguinte permanceu cara) ou então coroa (o ponteiro em uma coroa sendo a

Re: [obm-l] OBMEP 2021 - Fase 2 - N3

2021-11-09 Por tôpico Pedro Júnior
Obrigado, Ralph! Em ter., 9 de nov. de 2021 às 13:21, Ralph Costa Teixeira escreveu: > Suponho que (A) e (B) sejam fáceis -- basta seguir o algoritmo na mão e > ver o que acontece. > > Para facilitar a conversa, vou pensar em "tempo" como o número de > movimentos feitos... Ou seja, o tempo 0

Re: [obm-l] OBMEP 2021 - Fase 2 - N3

2021-11-09 Por tôpico Ralph Costa Teixeira
Suponho que (A) e (B) sejam fáceis -- basta seguir o algoritmo na mão e ver o que acontece. Para facilitar a conversa, vou pensar em "tempo" como o número de movimentos feitos... Ou seja, o tempo 0 corresponde à posição inicial; o tempo 1 seria logo após o primeiro movimento; etc. Para (C),

[obm-l] OBMEP 2021 - Fase 2 - N3

2021-11-09 Por tôpico Pedro Júnior
Olá pessoal, alguém aí conseguiu fazer essa questão da prova da OBMEP 2021 N3, fase 2? Se puder, ajuda aí... Valeu! 6) há 10 moedas em um círculo nomeadas de A a J, inicialmente todas com a face coroa virada para cima. No centro desse círculo, há um ponteiro que inicialmente aponta para a moeda

[obm-l] Análise

2021-11-06 Por tôpico Israel Meireles Chrisostomo
Olá, como posso provar que dado z_n uma sucessão de complexos tal que lim z_n=z então lim (1+z_n/n)^n=e^z? -- Esta mensagem foi verificada pelo sistema de antiv�rus e acredita-se estar livre de perigo.

Re: [obm-l] Lei dos cossenos e Lei dos senos

2021-10-15 Por tôpico Anderson Torres
Smells like argumentação circular. Em qui., 30 de set. de 2021 às 17:09, Israel Meireles Chrisostomo escreveu: > > Olá, ultimamente fiz uma prova para lei dos cossenos e senos, mas não sei se > está correta, alguém poderia por favor me ajudar na correção? > O link com a solução segue abaixo >

[obm-l] Lei dos cossenos e Lei dos senos

2021-09-30 Por tôpico Israel Meireles Chrisostomo
Olá, ultimamente fiz uma prova para lei dos cossenos e senos, mas não sei se está correta, alguém poderia por favor me ajudar na correção? O link com a solução segue abaixo https://www.overleaf.com/read/zfcqwwmgxnrt -- Esta mensagem foi verificada pelo sistema de antiv�rus e acredita-se estar

Re: [obm-l] geometria

2021-09-28 Por tôpico Israel Meireles Chrisostomo
Muito obrigado Em seg, 27 de set de 2021 21:25, Claudio Buffara escreveu: > O caso LLL de congruência implica que, dados 3 segmentos que obedecem aa > desigualdade triangular, o triângulo que os tem como lados é unicamente > determinado, a menos de uma isometria. > > Enviado do meu iPhone > > >

Re: [obm-l] geometria

2021-09-27 Por tôpico Claudio Buffara
O caso LLL de congruência implica que, dados 3 segmentos que obedecem aa desigualdade triangular, o triângulo que os tem como lados é unicamente determinado, a menos de uma isometria. Enviado do meu iPhone > Em 27 de set. de 2021, à(s) 19:50, Israel Meireles Chrisostomo > escreveu: > >  >

[obm-l] geometria

2021-09-27 Por tôpico Israel Meireles Chrisostomo
Olá pessoal. como faço para provar que o triângulo é um polígono rígido? Abraços, muito obrigado -- Israel Meireles Chrisostomo -- Esta mensagem foi verificada pelo sistema de antiv�rus e acredita-se estar livre de perigo.

Re: [obm-l] Conjuntos

2021-09-27 Por tôpico Pacini Bores
10% Em 26/09/2021 3:47, marcone augusto araújo borges escreveu: > Uma pessoa cética em relação às boas intenções da humanidade acredita que 70% > dos homens são violentos, 70% são desonestos e 70% são intolerantes. Se essa > pessoa estiver certa, em uma amostra ideal de 100 homens,

[obm-l] Conjuntos

2021-09-26 Por tôpico marcone augusto araújo borges
Uma pessoa cética em relação às boas intenções da humanidade acredita que 70% dos homens são violentos, 70% são desonestos e 70% são intolerantes. Se essa pessoa estiver certa, em uma amostra ideal de 100 homens, quantos são, no mínimo, simultaneamente desonestos, violentos e intolerantes? --

Re: [obm-l] lei dos senos

2021-09-25 Por tôpico Israel Meireles Chrisostomo
Muito obrigado Em sex, 24 de set de 2021 14:33, Claudio Buffara escreveu: > Se os ângulos do triângulo são dados, então o triângulo fica determinado a > menos de uma semelhança. > Daí, dado um lado, os outros ficam unicamente determinados, e > necessariamente obedecem à lei dos senos. > > Ou

Re: [obm-l] lei dos senos

2021-09-24 Por tôpico Claudio Buffara
Se os ângulos do triângulo são dados, então o triângulo fica determinado a menos de uma semelhança. Daí, dado um lado, os outros ficam unicamente determinados, e necessariamente obedecem à lei dos senos. Ou seja, dados a, b, c ângulos de um triângulo, e o lado de medida m, oposto ao ângulo a, os

[obm-l] Re: lei dos senos

2021-09-23 Por tôpico Israel Meireles Chrisostomo
Observação: eu não quero provar a lei dos senos, quero mostrar apenas a implicação apresentada.portanto se quiserem podem usar a lei dos senos para provar esse resultado Em sex., 24 de set. de 2021 às 00:54, Israel Meireles Chrisostomo < israelmchrisost...@gmail.com> escreveu: > Olá pessoal, eu

[obm-l] lei dos senos

2021-09-23 Por tôpico Israel Meireles Chrisostomo
Olá pessoal, eu estava resolvendo um problema daí então surgiu uma dúvida.A dúvida é a seguinte: sejam a,b,c ângulos de um triângulo e m,n,o lados de um triângulo qualquer , como provar que se m/sen(a)= n/sen(b)= o/sen(c) então a,b,c e m,n,o pertencem ao mesmo triângulo. ou seja vale a lei dos

[obm-l] Re: [obm-l] Re: [obm-l] Re: [obm-l] Re: matemática discreta

2021-09-20 Por tôpico Anderson Torres
Não consegui entender esse texto. Em seg., 20 de set. de 2021 às 22:37, Israel Meireles Chrisostomo escreveu: > > Obrigado > > Em seg, 20 de set de 2021 22:00, Israel Meireles Chrisostomo > escreveu: >> >> Tome n maior que n >> >> Em seg, 20 de set de 2021 20:49, Marcelo Salhab Brogliato >>

[obm-l] Re: [obm-l] Re: [obm-l] Re: matemática discreta

2021-09-20 Por tôpico Israel Meireles Chrisostomo
Obrigado Em seg, 20 de set de 2021 22:00, Israel Meireles Chrisostomo < israelmchrisost...@gmail.com> escreveu: > Tome n maior que n > > Em seg, 20 de set de 2021 20:49, Marcelo Salhab Brogliato < > msbro...@gmail.com> escreveu: > >> Oi Israel, >> >> Não consegui entender a questão. >> >>

[obm-l] Re: [obm-l] Re: [obm-l] Re: matemática discreta

2021-09-20 Por tôpico Israel Meireles Chrisostomo
Tome n maior que n Em seg, 20 de set de 2021 20:49, Marcelo Salhab Brogliato < msbro...@gmail.com> escreveu: > Oi Israel, > > Não consegui entender a questão. > > Exemplo: > > n = 10, m = 3, Fib(10 - 3 + 1) = Fib(8) = 21 > > (alpha**(2*n)) / (alpha**(n - m)) = alpha**(n + m) = 521.0019193787257

[obm-l] Re: [obm-l] Re: matemática discreta

2021-09-20 Por tôpico Marcelo Salhab Brogliato
Oi Israel, Não consegui entender a questão. Exemplo: n = 10, m = 3, Fib(10 - 3 + 1) = Fib(8) = 21 (alpha**(2*n)) / (alpha**(n - m)) = alpha**(n + m) = 521.0019193787257 Pela sua igualdade, alpha**(n + m) deveria ser 1/21, correto? Abraços, Marcelo Il giorno lun 20 set 2021 alle ore 15:54

[obm-l] Re: matemática discreta

2021-09-20 Por tôpico Israel Meireles Chrisostomo
já tentei de tudo, por favor me ajudem. Em seg., 20 de set. de 2021 às 19:39, Israel Meireles Chrisostomo < israelmchrisost...@gmail.com> escreveu: > Alguém poderia resolver o problema no link abaixo? > > >

[obm-l] matemática discreta

2021-09-20 Por tôpico Israel Meireles Chrisostomo
Alguém poderia resolver o problema no link abaixo? https://mathoverflow.net/questions/404417/alpha2n-fracf-n-m1-alphan-m-1-how-to-prove-that-equality-is-true -- Israel Meireles Chrisostomo -- Esta mensagem foi verificada pelo sistema de antiv�rus e acredita-se estar livre de perigo.

[obm-l] Re: [obm-l] Re: [obm-l] cálculo

2021-09-15 Por tôpico Israel Meireles Chrisostomo
Muito obrigado Em qua, 15 de set de 2021 11:36, Esdras Muniz escreveu: > O ponto é que tanto o conjunto dos números racionais quanto o conjunto dos > números irracionais são densos em R. Portanto, para todo intervalo não > degenerado, o máximo de f será 1 e o mínimo de f será zero. Daí, a

[obm-l] Re: [obm-l] cálculo

2021-09-15 Por tôpico Esdras Muniz
O ponto é que tanto o conjunto dos números racionais quanto o conjunto dos números irracionais são densos em R. Portanto, para todo intervalo não degenerado, o máximo de f será 1 e o mínimo de f será zero. Daí, a integral superior será sempre maior que a integral inferior, portanto a função não é

[obm-l] Re: [obm-l] cálculo

2021-09-15 Por tôpico Pedro Angelo
A definição de integrabilidade Riemann passa por verificar que, para partições P suficientemente finas, a soma superior S(f;P) é parecida com a soma inferior s(f;P). Faça o que sempre deve ser feito nesse tipo de problema: calcule exemplos concretos. Escolha partições quaisquer (pequenas, pois vc

[obm-l] cálculo

2021-09-14 Por tôpico Israel Meireles Chrisostomo
Olá pessoal. eu estou me esforçando para entender esse exemplo do guidorizzi, alguém poderia me explicar?Aqui vai: Seja f uma função, tal que se x é racional então f igual a 1, se x é irracional então f igual a zero. Mostre que a função não é riemann integrável. -- Israel Meireles Chrisostomo

Fwd: [obm-l]

2021-09-06 Por tôpico Eric Campos Bastos Guedes
Bastos Guedes Date: sáb., 4 de set. de 2021 00:33 Subject: [obm-l] To: Olá a todos. Gostaria de pedir licença para que vocês avaliem um algoritmo que eu fiz para fatorar números grandes com fatores primos também grandes. Eu acredito que esse algoritmo quebre o RSA O algoritmo é o seguinte: PASSO

[obm-l]

2021-09-03 Por tôpico Eric Campos Bastos Guedes
Olá a todos. Gostaria de pedir licença para que vocês avaliem um algoritmo que eu fiz para fatorar números grandes com fatores primos também grandes. Eu acredito que esse algoritmo quebre o RSA O algoritmo é o seguinte: PASSO 1: faça A=3 PASSO 2: N é o inteiro a ser fatorado PASSO 3: M = N**16

[obm-l] OCM 2007 problema 4

2021-08-28 Por tôpico qedtexte
Sauda,c~oes, OCM = Cear​ Seja 0 = x = 1. Prove que (1+x)^n + (1-x)^n = 2(1+x^{n/(n-1)})^{n-1} para n = 2. D para provar por Induo ? Lus -- Esta mensagem foi verificada pelo sistema de antiv�rus e acredita-se estar livre de perigo.

[obm-l] Re: [obm-l] Ajuda numa questão da OBM 1987

2021-08-21 Por tôpico Anderson Torres
Em ter., 20 de jul. de 2021 às 18:25, Prof. Douglas Oliveira escreveu: > > Tem-se um bolo em forma de prisma triangular, cuja base está em um plano > horizontal. Dois indivíduos vão dividir o bolo de acordo com a seguinte > regra: o primeiro escolhe um ponto na base superior do bolo e o segundo

[obm-l] Re: [obm-l] "números biquadrados"

2021-08-13 Por tôpico Anderson Torres
Em qui, 12 de ago de 2021 21:17, marcone augusto araújo borges < marconeborge...@hotmail.com> escreveu: > 1233 = 12^2 + 33^2 > Em uma prova da bom nível 2, o número 1233 foi apresentado como > "biquadrado" e foi pedido outro número biquadrado > Eu pensei > A^2+ B^2 = 100A + B > A^2 - 100A + B^2 -

[obm-l] "números biquadrados"

2021-08-12 Por tôpico marcone augusto araújo borges
1233 = 12^2 + 33^2 Em uma prova da bom nível 2, o número 1233 foi apresentado como "biquadrado" e foi pedido outro número biquadrado Eu pensei A^2+ B^2 = 100A + B A^2 - 100A + B^2 - B = 0 Seriam dois valores para A cuja soma é 100, então se um deles é 12 o outro é 88 Observei que 8833 = 88^2 +

[obm-l] Re: [obm-l] Re: [obm-l] Álgebra

2021-07-27 Por tôpico Anderson Torres
Em dom., 25 de jul. de 2021 às 15:23, Ralph Costa Teixeira escreveu: > > Sem ser muito formal: (a,b) e (c,d) sao dois vetores do plano, unitários e > ortogonais. Ou seja, um deles eh igual ao outro girado de 90 graus. Assim > (c,d)=(-b,a) ou (c,d)=(b,-a). De um jeito ou de outro, cd=-ab, ou

[obm-l] Re: [obm-l] Re: [obm-l] Álgebra

2021-07-25 Por tôpico Pacini Bores
Vi também assim : (ac+bd)(ad+bc) = cd(a^2+b^2)+ab(c^2+d^2). 0= cd.1 + ab.1, logo ab+cd =0. É claro que a solução do Ralph é mais elegante... Abraços Pacini Em 25/07/2021 15:10, Ralph Costa Teixeira escreveu: > Sem ser muito formal: (a,b) e (c,d) sao dois vetores do plano,

[obm-l] RE: [obm-l] Novos avanços sobre a Hipótese do Continuum

2021-07-25 Por tôpico Paulo Santa Rita
Oi Bouskela e demais membros desta lista... obm-l ! No mínimo interessante... Na verdade, dizer que a cardinalidade do contínuo é "C" é apenas uma convenção e demonstração de ignorância, pois não sabemos (ainda) a que álefe da sequência do Cantor este "C" corresponde...

[obm-l] Re: [obm-l] Álgebra

2021-07-25 Por tôpico Ralph Costa Teixeira
Sem ser muito formal: (a,b) e (c,d) sao dois vetores do plano, unitários e ortogonais. Ou seja, um deles eh igual ao outro girado de 90 graus. Assim (c,d)=(-b,a) ou (c,d)=(b,-a). De um jeito ou de outro, cd=-ab, ou seja, resposta 0. On Sun, Jul 25, 2021 at 10:03 AM marcone augusto araújo borges <

[obm-l] Álgebra

2021-07-25 Por tôpico marcone augusto araújo borges
a, b, c, d são números reais tais que a^2+b^2 = c^2 + d^2 = 1, ac + bd = 0. Calcule ab + cd Desde já agradeço -- Esta mensagem foi verificada pelo sistema de antivírus e acredita-se estar livre de perigo.

[obm-l] Re: [obm-l] Ajuda numa questão da OBM 1987

2021-07-20 Por tôpico joao pedro b menezes
Eu pensaria em trabalhar com os pontos notáveis, talvez o baricentro, e argumentar que em qualquer outro ponto é possível realizar um corte que o prejudique mais. Isso é só uma teoria e, portanto, é possível que esteja totalmente errada. -- Esta mensagem foi verificada pelo sistema de antiv�rus

[obm-l] Ajuda numa questão da OBM 1987

2021-07-20 Por tôpico Prof. Douglas Oliveira
*Tem-se um bolo em forma de prisma triangular, cuja base está em um plano horizontal. Dois indivíduos vão dividir o bolo de acordo com a seguinte regra: o primeiro escolhe um ponto na base superior do bolo e o segundo corta o bolo por um plano vertical à sua escolha, passando porém pelo ponto

[obm-l] Novos avanços sobre a Hipótese do Continuum

2021-07-18 Por tôpico bouskela
Recebi da Quanta Magazine o artigo identificado a seguir: How Many Numbers Exist? Infinity Proof Moves Math Closer to an Answer For 50 years, mathematicians have believed that the total number of real numbers is unknowable. A new proof suggests otherwise. Este artigo foi escrito por

[obm-l] solicitação para participação no grupo

2021-07-16 Por tôpico jeffma...@yahoo.com.br
Olá! Solicito permissão para participar do grupoAttJefferson -- Esta mensagem foi verificada pelo sistema de antiv�rus e acredita-se estar livre de perigo.

Re: [obm-l] Limites

2021-06-30 Por tôpico Anderson Torres
Em sex., 25 de jun. de 2021 às 23:38, Israel Meireles Chrisostomo escreveu: > > Alguém aí consegue provar o teorema do confronto?Em caso afirmativo por favor > prove-o > ?? > -- > Esta mensagem foi verificada pelo sistema de antivírus e > acredita-se estar livre de perigo. -- Esta mensagem

Re: [obm-l] Limites

2021-06-26 Por tôpico Israel Meireles Chrisostomo
m nome de > Israel Meireles Chrisostomo > *Enviado:* sexta-feira, 25 de junho de 2021 23:27 > *Para:* obm-l > *Assunto:* [obm-l] Limites > > Alguém aí consegue provar o teorema do confronto?Em caso afirmativo por > favor prove-o > > -- > Esta mensagem foi verifica

Re: [obm-l] Limites

2021-06-26 Por tôpico Maikel Andril Marcelino
De: owner-ob...@mat.puc-rio.br em nome de Israel Meireles Chrisostomo Enviado: sexta-feira, 25 de junho de 2021 23:27 Para: obm-l Assunto: [obm-l] Limites Alguém aí consegue provar o teorema do confronto?Em caso afirmativo por favor prove-o -- Esta mensagem

[obm-l] Limites

2021-06-25 Por tôpico Israel Meireles Chrisostomo
Alguém aí consegue provar o teorema do confronto?Em caso afirmativo por favor prove-o -- Esta mensagem foi verificada pelo sistema de antiv�rus e acredita-se estar livre de perigo.

Re: [obm-l] probabilidade condicional

2021-06-25 Por tôpico Daniel Jelin
Obrigado, Ralph! Em qui, 24 de jun de 2021 23:55, Ralph Costa Teixeira escreveu: > Sim, são falsas! > > Seu exemplo mata o problema! Seus eventos A e B são independentes, mas: > > Em (1), P (A | B e C) = 0, enquanto P(A | C) = 1/2. > Em (2), P(A e B | C) = 0, enquanto P(A | C) = P (B | C) =

Re: [obm-l] probabilidade condicional

2021-06-24 Por tôpico Ralph Costa Teixeira
Sim, são falsas! Seu exemplo mata o problema! Seus eventos A e B são independentes, mas: Em (1), P (A | B e C) = 0, enquanto P(A | C) = 1/2. Em (2), P(A e B | C) = 0, enquanto P(A | C) = P (B | C) = 1/2. Em suma, quando uma nova informação (C) chega, eventos (A) e (B) que eram independentes

[obm-l] probabilidade condicional

2021-06-24 Por tôpico Daniel Jelin
Caros, duas dúvidas elementares sobre probabilidade condicional, quem sabe possam me ajudar. Leio, em mais de um lugar, que: 1) Se A e B são independentes, então P(A | B e C) = P (A | C) A explicação parece fazer sentido: se A não depende de B, tanto que faz que B seja dado ou não. Em conexão

Re: [obm-l] Produto infinito do seno

2021-06-18 Por tôpico Israel Meireles Chrisostomo
Esse daqui(da imagem abaixo) [image: image.png] Livre de vírus. www.avast.com .

Re: [obm-l] Produto infinito do seno

2021-06-18 Por tôpico Israel Meireles Chrisostomo
falo do produto infinito do seno de euler Livre de vírus. www.avast.com .

[obm-l] Re: [obm-l] Re: [obm-l] Re: [obm-l] Re: [obm-l] Função

2021-05-29 Por tôpico Israel Meireles Chrisostomo
obrigado Livre de vírus. www.avast.com . <#DAB4FAD8-2DD7-40BB-A1B8-4E2AA1F9FDF2> Em qua., 19 de mai.

[obm-l] Re: [obm-l] base de numeração

2021-05-19 Por tôpico Anderson Torres
Em ter., 27 de abr. de 2021 às 17:40, Daniel Quevedo escreveu: > > Os oito últimos algarismos do número 27^1986 quando escrito na base 2 são: > a) 11011001 > b) 11011101 > c) 1001 > d) 11011011 > e) 10011001 > > gab: A Calcule o resto da divisão de 27^1986 por 2^8, depois converta para

[obm-l] Re: [obm-l] Re: [obm-l] Re: [obm-l] Função

2021-05-19 Por tôpico Anderson Torres
Em seg., 26 de abr. de 2021 às 17:18, Israel Meireles Chrisostomo escreveu: > > Mas aí então a+bi e b+ai são os mesmos números Não são. 4+5i e 5+4i são diferentes, e 4+5i < 5+4i por essas regras. > > Em seg, 26 de abr de 2021 13:36, Anderson Torres > escreveu: >> >> Em qui., 22 de abr. de

Re: [obm-l] Produto infinito do seno

2021-05-19 Por tôpico Anderson Torres
Em seg., 17 de mai. de 2021 às 18:58, Israel Meireles Chrisostomo escreveu: > > Alguém aí sabe quantas provas existem para se verificar q d fato o produto > infinito do seno é verdadeiro?Eu tenho uma. > HEIN > -- > Esta mensagem foi verificada pelo sistema de antivírus e > acredita-se

[obm-l] Produto infinito do seno

2021-05-17 Por tôpico Israel Meireles Chrisostomo
Alguém aí sabe quantas provas existem para se verificar q d fato o produto infinito do seno é verdadeiro?Eu tenho uma. -- Esta mensagem foi verificada pelo sistema de antiv�rus e acredita-se estar livre de perigo.

[obm-l] Olimpíada Universitária Fase 2

2021-05-13 Por tôpico João
Olá. Sobre as provas que aconteceram em março. É que eu estava olhando e queria saber mais. Foi a primeira edição este ano? Sendo objetivo, enumerando por temas e assuntos a serem estudados, questão a questão, como ficaria um roteiro de estudos daquelas seis questões por assunto? -- Esta

Re: [obm-l] f(x + y) = f(x) + f(y)

2021-05-10 Por tôpico Bruno Visnadi
A princípio, não há nada que garanta que f seja derivável ou mesmo que o limite exista para esta prova valer. Mas, de fato, se o domínio está restrito a Q, você pode mostrar que f(x) = ax para algum a. Um caminho é definir f(1) = a e mostrar que f(1/n) = a/n, para então chegar em f(m/n) = ma/n.

Assunto: Re: [obm-l] f(x + y) = f(x) + f(y)

2021-05-05 Por tôpico Artur Costa Steiner
Se vc adicionar a hipótese de que f é contínua em algum real x0, a conclusão desejada torna-se válida. Se vc quiser elocubrar um pouco, pode seguir os seguintes passos,: Mostre que continuidade em x0 implica continuidade em 0 que, por sua vez, implica continuidade em toda a reta real. Mostre que

Re: [obm-l] f(x + y) = f(x) + f(y)

2021-05-05 Por tôpico Claudio Buffara
f(x) = ax + b só satisfaz isso se b = 0. Tente com x+1, por exemplo. E mais: sem alguma outra condição (do tipo continuidade ou monotonicidade) ainda assim a expressão não implica que f(x) = ax. Abs, Cláudio. Enviado do meu iPhone > Em 5 de mai. de 2021, à(s) 09:13, joao pedro b menezes >

[obm-l] f(x + y) = f(x) + f(y)

2021-05-05 Por tôpico joao pedro b menezes
Eu estava fazendo um exercício de equações funcionais e me deparei com essa expressão. Não sei o que aconteceu, mas tive uma crise existencial e decidi provar que isso implica f(x) = ax + b( ou pelo menos acho que implica). Essa prova estaria certa?: (obs: a função é definida nos racionais) f(x +

[obm-l] RES: [obm-l] Livros OBM - Universitária

2021-04-30 Por tôpico bouskela
realizadas. Albert Bouskelá <mailto:bousk...@gmail.com> bousk...@gmail.com De: owner-ob...@mat.puc-rio.br Em nome de Wallace Albert Enviada em: terça-feira, 27 de abril de 2021 20:49 Para: obm-l@mat.puc-rio.br Assunto: [obm-l] Livros OBM - Universitária Pessoal, bom dia! Pr

[obm-l] Livros OBM - Universitária

2021-04-27 Por tôpico Wallace Albert
Pessoal, bom dia! Pretendo iniciar meus estudos pra OBM nível U em breve e gostaria de saber se vocês têm alguma recomendação de livros didáticos para que eu possa estudar. No site da OBM vejo que o foco é mais voltado às olimpíadas de ensino médio/internacionais. Alguém poderia me ajudar? Desde

[obm-l] base de numeração

2021-04-27 Por tôpico Daniel Quevedo
Os oito últimos algarismos do número 27^1986 quando escrito na base 2 são: a) 11011001 b) 11011101 c) 1001 d) 11011011 e) 10011001 gab: A -- Fiscal: Daniel Quevedo -- Esta mensagem foi verificada pelo sistema de antiv�rus e acredita-se estar livre de perigo.

[obm-l] Re: [obm-l] Re: [obm-l] Função

2021-04-26 Por tôpico Israel Meireles Chrisostomo
Mas aí então a+bi e b+ai são os mesmos números Em seg, 26 de abr de 2021 13:36, Anderson Torres < torres.anderson...@gmail.com> escreveu: > Em qui., 22 de abr. de 2021 às 07:19, Israel Meireles Chrisostomo > escreveu: > > > > Me desculpem se eu estou falando bobagem, mas considere uma função

[obm-l] Re: [obm-l] Função

2021-04-26 Por tôpico Anderson Torres
Em qui., 22 de abr. de 2021 às 07:19, Israel Meireles Chrisostomo escreveu: > > Me desculpem se eu estou falando bobagem, mas considere uma função com > domínio complexo, então essa função não pode ser bijetora, pois toda função > bijetora ou é crescente ou é decrescente, mas não há ordem nos

Re: [obm-l]

2021-04-26 Por tôpico Anderson Torres
Em dom., 25 de abr. de 2021 às 14:34, Artur Costa Steiner escreveu: > > Oh, no meu email anterior, onde se > lê raiz(3), leia-se raiz_cúbica(2). Tô fazendo um tratamento na vista e ando > com dificuldade para digitar num celular. > Um cara de 69 anos como eu não deveria mais participar deste

Re: [obm-l] Magnitude

2021-04-26 Por tôpico Anderson Torres
Em seg., 26 de abr. de 2021 às 00:16, Pedro Lazéra escreveu: > > Boa noite, Maikel. > > A quantidade de algarismos de um número x (na base 10) é "1 + piso de > log(x)", em que "log" é a função logaritmo na base 10. Você pode verificar > isso assim: 10^n, para n inteiro >= 0, é o menor número do

Re: [obm-l] Magnitude

2021-04-25 Por tôpico Pedro Lazéra
Boa noite, Maikel. A quantidade de algarismos de um número x (na base 10) é "1 + piso de log(x)", em que "log" é a função logaritmo na base 10. Você pode verificar isso assim: 10^n, para n inteiro >= 0, é o menor número do mundo com n+1 algarismos. Além disso, log(10^n) = n. Por fim, log é uma

Re: [obm-l]

2021-04-25 Por tôpico Artur Costa Steiner
Oh, no meu email anterior, onde se lê raiz(3), leia-se raiz_cúbica(2). Tô fazendo um tratamento na vista e ando com dificuldade para digitar num celular. Um cara de 69 anos como eu não deveria mais participar deste grupo Artur Em dom., 25 de abr. de 2021 14:16, Artur Costa Steiner <

Re: [obm-l]

2021-04-25 Por tôpico Artur Costa Steiner
raiz(2)) e raiz(3) são inteiros algébricos, visto serem raízes de x^2 - 2 e x^3 - 2, respectivamente. Segundo um clássico teorema da Teoria dos Números, a soma de dois inteiros algébricos é inteira algébrica. E um inteiro algébrico é racional se, e somente se, for inteiro. Como, conforme já

[obm-l] Re: [obm-l] RES: [obm-l] Re: [obm-l] INFLAÇÂO MÁXIMA

2021-04-24 Por tôpico Pedro Júnior
er-ob...@mat.puc-rio.br *Em nome de > *Daniel Jelin > *Enviada em:* sexta-feira, 23 de abril de 2021 12:30 > *Para:* obm-l@mat.puc-rio.br > *Assunto:* [obm-l] Re: [obm-l] INFLAÇÂO MÁXIMA > > > > Curioso, pra mim deu muito perto, 17,6470...% > > Resolvi a seguinte inequ

Re: [obm-l] Probabilidade

2021-04-23 Por tôpico Pacini Bores
Obrigado Ralph pela explicação didática. Ficou esclarecida a minha dúvida Abraços Pacini Em 23/04/2021 16:59, Ralph Costa Teixeira escreveu: > Ah, Pacini, você levanta um ponto interessante... > > Primeiro, deixa eu esclarecer: eu usei p(n) = Pr (A vai vencer o jogo | A tem > n

Re: [obm-l]

2021-04-23 Por tôpico Marcos Martinelli
Legal, Matheus. Minha ideia foi encontrar um polinômio em m.n (m = raiz(2) e n=raiz_cúbica(2)) de coeficientes racionais. Pra isso desenvolvi m^k + n^k (k >= 0) até k=6 e encontrei um de grau 6 com coeficientes dependendo só de m+n. Se m+n for racional, usei o fato de se a + beta (a racional e

[obm-l] Re: [obm-l] Re: [obm-l] Função

2021-04-23 Por tôpico Israel Meireles Chrisostomo
Obrigado Em qui, 22 de abr de 2021 11:25, Artur Costa Steiner < artur.costa.stei...@gmail.com> escreveu: > O que vc disse só vale para funções contínuas de R em R. No domínio > complexo, não vale. > Nos complexos, uma função inteira é injetora se, e somente se, for um > mapeamento afim não

Re: [obm-l]

2021-04-23 Por tôpico Matheus Secco
Oi, Marcos. Não é difícil verificar que raiz(2) + raiz_cubica(2) é uma raiz do polinômio x^6 - 6 x^4 - 4 x^3 + 12 x^2 - 24 x - 4. Com isso, pelo teorema das raízes racionais, se raiz(2) + raiz_cubica(2) fosse racional, teria que ser um inteiro e é fácil verificar que 2 < raiz(2) + raiz_cubica(2) <

Re: [obm-l] Probabilidade

2021-04-23 Por tôpico Ralph Costa Teixeira
Ah, Pacini, você levanta um ponto interessante... Primeiro, deixa eu esclarecer: eu usei p(n) = Pr (A vai vencer o jogo | A tem n pontos a mais do que B agora); ou seja, não seria exatamente o que você interpretou ali. Daqui meu argumento de simetria: a partir do momento em que A tem 0 pontos a

[obm-l]

2021-04-23 Por tôpico Marcos Martinelli
Opa, pessoal. Pensei nos últimos dias no problema seguinte. Cheguei a uma solução um pouco mais genérica, mas me deu trabalho. Gostaria de estudar outras abordagens. Problema) Prove que raiz (2) + raiz_cúbica (2) é irracional. Na sequência posto um rascunho do que pensei. Obrigado. -- Esta

[obm-l] RES: [obm-l] Re: [obm-l] INFLAÇÂO MÁXIMA

2021-04-23 Por tôpico bouskela
usk...@gmail.com> bousk...@gmail.com De: owner-ob...@mat.puc-rio.br Em nome de Daniel Jelin Enviada em: sexta-feira, 23 de abril de 2021 12:30 Para: obm-l@mat.puc-rio.br Assunto: [obm-l] Re: [obm-l] INFLAÇÂO MÁXIMA Curioso, pra mim deu muito perto, 17,6470...% Resolvi a seguinte inequação, com

Re: [obm-l] Probabilidade

2021-04-23 Por tôpico Pacini Bores
Desculpe Ralph, O que não ficou claro pra mim foi o fato de que p(0) =1/2 , já que p(0) traduz a probabilidade de de ficar com diferença de zero ponto agora ou depois, ou seja, partindo de zero ponto de diferença entre os dois jogadores, poderia ficar assim a vida toda, não ? Em que estou

[obm-l] Re: [obm-l] INFLAÇÂO MÁXIMA

2021-04-23 Por tôpico Daniel Jelin
Curioso, pra mim deu muito perto, 17,6470...% Resolvi a seguinte inequação, com x = 1 + (inflação): 1.1*1000x - (1.1*1000x - 1000)*0.4>=1000x 1.1 x - 0.44 x + 0.4 >= x x<=0.4/0.34= 1.176470... Parece simples. O que tá escapando aqui? On Fri, Apr 23, 2021 at 11:23 AM Pedro Júnior wrote: > Olá

[obm-l] INFLAÇÂO MÁXIMA

2021-04-23 Por tôpico Pedro Júnior
Olá pessoal, acabei me enrolando nesse probleminha da Olimpíada Brasileira de Economia. Será que alguém pode me ajudar? Vai junto o gabarito da competição, isso foi em 2020. *01)* Um título comprado por mil reais promete pagar juros reais de 10% a.a. A alíquota de imposto é de 40%. Qual a

[obm-l] Re: [obm-l] Função

2021-04-22 Por tôpico Artur Costa Steiner
O que vc disse só vale para funções contínuas de R em R. No domínio complexo, não vale. Nos complexos, uma função inteira é injetora se, e somente se, for um mapeamento afim não constante, caso em que é bijetora. Artur Em qui., 22 de abr. de 2021 07:19, Israel Meireles Chrisostomo <

[obm-l] Re: [obm-l] Função

2021-04-22 Por tôpico Eduardo Henrique Rodrigues do Nascimento
Cara, toda função real contínua e bijetora é monótona. Como contraexemplo se f não for contínua: x+1 para x no intervalo [0,1[ f(x)={x, para x≥2 e x<0 x-1 para x no intervalo [1,2[ então f não é crescente em todo o seu domínio: 1/2<3/2; mas f(1/2)=3/2>1/2=f(3/2). além

[obm-l] Função

2021-04-22 Por tôpico Israel Meireles Chrisostomo
Me desculpem se eu estou falando bobagem, mas considere uma função com domínio complexo, então essa função não pode ser bijetora, pois toda função bijetora ou é crescente ou é decrescente, mas não há ordem nos complexos -- Esta mensagem foi verificada pelo sistema de antiv�rus e acredita-se

[obm-l] Re: [obm-l] Re: [obm-l] Questão sobre desigualdades

2021-04-14 Por tôpico Carlos Monteiro
De onde saiu essa desigualdade? Em qua., 14 de abr. de 2021 às 20:39, Anderson Torres < torres.anderson...@gmail.com> escreveu: > Em qua., 14 de abr. de 2021 às 15:54, Carlos Monteiro > escreveu: > > > > Encontre os valores máximo e mínimo da expressão: x/(x^2+1) + y/(y^2+1) > + z/(z^2+1) ,

[obm-l] Re: [obm-l] Questão sobre desigualdades

2021-04-14 Por tôpico Anderson Torres
Em qua., 14 de abr. de 2021 às 15:54, Carlos Monteiro escreveu: > > Encontre os valores máximo e mínimo da expressão: x/(x^2+1) + y/(y^2+1) + > z/(z^2+1) , onde x, y e z são números reais que satisfazem x+y+z = 1. > > Verifica-se que 3(12x+1)/50 >= x/(x^2+1), e assim o valor máximo é 3/10 > >

[obm-l] Questão sobre desigualdades

2021-04-14 Por tôpico Carlos Monteiro
Encontre os valores máximo e mínimo da expressão: x/(x^2+1) + y/(y^2+1) + z/(z^2+1) , onde x, y e z são números reais que satisfazem x+y+z = 1. -- Esta mensagem foi verificada pelo sistema de antiv�rus e acredita-se estar livre de perigo.

Re: [obm-l] transcendencia

2021-04-10 Por tôpico Anderson Torres
Em qui., 1 de abr. de 2021 às 18:02, Israel Meireles Chrisostomo escreveu: > > > Como posso provar que se u é um número transcendente e a_k são números > algébricos quaisquer, para todo k natural, então ua_0+ ua_1+ ua_2+...+ ua_n > não pode ser igual a zero. Fatorando U. > -- > Israel

Re: [obm-l] Magnitude

2021-04-10 Por tôpico Anderson Torres
Em sáb., 3 de abr. de 2021 às 01:13, Maikel Andril Marcelino escreveu: > > Quantos algarismos tem o número (100!) ? Em outras palavras, qual é o log(100!)/log(10). O Google me diz que isso é 157,97 - logo, 158 dígitos. > > > Atenciosamente, > > Maikel Andril Marcelino > Assistente de Aluno -

[obm-l] Re: [obm-l] Algébricos

2021-04-10 Por tôpico Anderson Torres
Em seg., 5 de abr. de 2021 às 21:57, Israel Meireles Chrisostomo escreveu: > > O número i é algebricamente dependente de pi? > O que é algebricamente dependente? > -- > Esta mensagem foi verificada pelo sistema de antivírus e > acredita-se estar livre de perigo. -- Esta mensagem foi

Re: [obm-l] Probabilidade

2021-04-09 Por tôpico Ralph Costa Teixeira
; resultados possíveis numa determinada rodada do jogo! Dito assim, o >> enunciado admite, para cada rodada 4 possibilidades: (A=1, B=1); (A=1, >> B=0); (A=0, B=1); (A=0, B=0). >> >> >> >> *Albert Bouskelá* >> >> bousk...@gmail.com >

Re: [obm-l] Probabilidade

2021-04-09 Por tôpico Daniel Jelin
(A=1, > B=0); (A=0, B=1); (A=0, B=0). > > > > *Albert Bouskelá* > > bousk...@gmail.com > > > > *De:* owner-ob...@mat.puc-rio.br *Em nome de > *Professor Vanderlei Nemitz > *Enviada em:* quinta-feira, 8 de abril de 2021 14:34 > *Para:* OBM > *Assunto:* R

Re: [obm-l] Probabilidade

2021-04-09 Por tôpico Pacini Bores
Acredito que foi este ano. Passaram pra mim desta forma. Pacini Em 08/04/2021 14:33, Professor Vanderlei Nemitz escreveu: > Muito legal esse tipo de problema. > Em que ano caiu, você sabe, Pacini? > > Em sáb., 3 de abr. de 2021 às 15:22, Pacini Bores > escreveu: > >> Olá pessoal,

RES: [obm-l] Probabilidade

2021-04-08 Por tôpico bouskela
); (A=0, B=0). Albert Bouskelá <mailto:bousk...@gmail.com> bousk...@gmail.com De: owner-ob...@mat.puc-rio.br Em nome de Professor Vanderlei Nemitz Enviada em: quinta-feira, 8 de abril de 2021 14:34 Para: OBM Assunto: Re: [obm-l] Probabilidade Muito legal esse tipo de problema.

Re: [obm-l] Probabilidade

2021-04-08 Por tôpico Professor Vanderlei Nemitz
Muito legal esse tipo de problema. Em que ano caiu, você sabe, Pacini? Em sáb., 3 de abr. de 2021 às 15:22, Pacini Bores escreveu: > Olá pessoal, Encontrei uma resposta que não está entre as opções desta > questão do Canguru. > > " um certo jogo tem um vencedor quando este atinge 3 pontos a

Re: [obm-l] Magnitude

2021-04-07 Por tôpico qedtexte
Sauda,c~oes, Com o WA, 158. Lus Data: 03/04/2021 De: Maikel Andril Marcelino maikel.marcel...@ifrn.edu.br Para: obm-l@mat.puc-rio.br Assunto: [obm-l] Magnitude Quantos algarismos tem o nmero (100!) ? Atenciosamente, Maikel Andril Marcelino Assistente de Aluno - Biblioteca - Ramal

[obm-l] Algébricos

2021-04-05 Por tôpico Israel Meireles Chrisostomo
O número i é algebricamente dependente de pi? -- Esta mensagem foi verificada pelo sistema de antiv�rus e acredita-se estar livre de perigo.

Re: [obm-l] Probabilidade

2021-04-03 Por tôpico Pacini Bores
Obrigado Ralph Abraços Em 03/04/2021 18:08, Ralph Costa Teixeira escreveu: > Vou dizer que "o jogo está na posicao n" quando A tem n pontos de vantagem; e > vou chamar de p(n) a probabilidade de A vencer o jogo (agora ou depois) > sabendo que (agora) A tem n pontos mais do que B. > >

[obm-l] Probabilidade

2021-04-03 Por tôpico Pacini Bores
Olá pessoal, Encontrei uma resposta que não está entre as opções desta questão do Canguru. " um certo jogo tem um vencedor quando este atinge 3 pontos a frente do oponente. Dois jogadores A e B estão jogando e, num determinado momento, A está 1 ponto a frente de B. Os jogadores têm

Re: [obm-l] Probabilidade

2021-04-03 Por tôpico Ralph Costa Teixeira
Vou dizer que "o jogo está na posicao n" quando A tem n pontos de vantagem; e vou chamar de p(n) a probabilidade de A vencer o jogo (agora ou depois) sabendo que (agora) A tem n pontos mais do que B. Por exemplo, p(3)=1, p(-3)=0 e p(0)=1/2 (por simetria). Aliás, por simetria, vemos que

<    1   2   3   4   5   6   7   8   9   10   >